Talk:2017 AMC 12A Problems/Problem 20

Revision as of 16:22, 8 February 2017 by Rajat.mittal (talk | contribs) (Created page with "The solution says the answer is D, but it doesn't account for the negative 2016th-root of 2017, because that will still have correct, desired solutions for (a, b). Based on th...")
(diff) ← Older revision | Latest revision (diff) | Newer revision → (diff)

The solution says the answer is D, but it doesn't account for the negative 2016th-root of 2017, because that will still have correct, desired solutions for (a, b). Based on that, the answer would be E - 597. Am I seeing something wrong here?